-
Notifications
You must be signed in to change notification settings - Fork 0
/
Copy pathlecture4.tex
132 lines (101 loc) · 5.3 KB
/
lecture4.tex
1
2
3
4
5
6
7
8
9
10
11
12
13
14
15
16
17
18
19
20
21
22
23
24
25
26
27
28
29
30
31
32
33
34
35
36
37
38
39
40
41
42
43
44
45
46
47
48
49
50
51
52
53
54
55
56
57
58
59
60
61
62
63
64
65
66
67
68
69
70
71
72
73
74
75
76
77
78
79
80
81
82
83
84
85
86
87
88
89
90
91
92
93
94
95
96
97
98
99
100
101
102
103
104
105
106
107
108
109
110
111
112
113
114
115
116
117
118
119
120
121
122
123
124
125
126
127
128
129
130
131
132
\section{Lecture 4: Lagrange Multipliers, Near Equilibrium Dynamics \& Oscillators}
\subsection{Lagrange Multipliers: Constrained Optimization}
Our standard Lagrangian approach requires expressing the positions of all particles in
terms of generalized coordinates, $r_\alpha\left(q_i, t\right)$. This can be complex,
making it difficult to obtain an explicit form for the Lagrangian,
$L\left(q_i, \dot{q_i}, t\right)$. A natural question arises: can we formulate mechanics
directly in terms of the particle positions, $\mathbf{r_\alpha}$, and impose constraints
directly at the level of the equations of motion, rather than at the level of the action?
This is where Lagrange multipliers become a valuable tool.
\subsubsection{Review of Lagrange Multipliers in Calculus}
Let's first recall how Lagrange multipliers work in standard calculus. Suppose we want to
minimize a function $F\left(x, y\right)$. Without constraints, this involves simply
finding where the partial derivatives vanish:
\begin{equation}
\frac{\partial F}{\partial x} = \frac{\partial F}{\partial y} = 0
\end{equation}
However, if we want to minimize $F\left(x, y\right)$ subject to a constraint given by
$g\left(x, y\right) = 0$, we introduce a Lagrange multiplier, $\lambda$. We then
minimize the modified function: $F\left(x, y\right) + \lambda \ g\left(x, y\right)$. This
leads to the following set of equations:
\begin{align}
\frac{\partial F}{\partial x} + \lambda \ \frac{\partial g}{\partial x} &= 0 \\
\frac{\partial F}{\partial y} + \lambda \ \frac{\partial g}{\partial y} &= 0 \\
g\left(x, y\right) &= 0
\end{align}
Notice that we now have three equations for three unknowns: $x$, $y$, and $\lambda$. The
first two equations define the stationary points of $F$ subject to the constraint, and
the third equation ensures that the constraint is satisfied. This method provides a
systematic way to solve constrained optimization problems.
\subsubsection{Lagrange Multipliers in the Calculus of Variations}
The same principle can be extended to the calculus of variations. Let's say we have $M$
functions $y_i\left(x\right), i = 1, 2, \cdots, M$, and we want to minimize an action
integral:
\begin{equation}
I[y_i\left(x\right)] = \int dx \ F\left(y_i, \dot{y_i}, x\right)
\end{equation}
Furthermore, suppose we have $M-N$ constraints of the form $G_k[y_i] = 0, k = 1, 2, \cdots, M - N$.
The traditional approach would be to express all $M$ functions, $y_i$, in terms of $N$
independent degrees of freedom, however, we can avoid this by using Lagrange multipliers.
To incorporate the constraints, we modify the action integral as follows:
\begin{equation}
\tilde{I}[y_i, \lambda_k] = \int dx \ \left(F\left(y_i, \dot{y_i}, x\right) + \sum_k \lambda_k \left(x\right) G_k[y_i]\right)
\end{equation}
Here, $\lambda_k(x)$ are now functions of $x$, the independent variable of the integral.
We treat $\tilde{I}$ as the functional we want to minimize, and obtain its Euler-Lagrange
equations:
\begin{align}
\frac{\delta \tilde{I}}{\delta y_i} = \frac{\delta I}{\delta y_i} + \sum_k \lambda_k \frac{\delta G_k}{\delta y_i} &= 0 \\
G_k[y_i] &= 0
\end{align}
This gives us a total of $M + (M - N)$ equations. We have $M$ equations from the
functional derivative with respect to each $y_i$, plus $M-N$ equations from the
constraints. These equations involve the $M$ unknowns $y_i$ and the $M-N$ Lagrange
multiplier functions, $\lambda_k(x)$. These can, in principle, be solved to obtain the
solution to the constrained variational problem.
\begin{example}
Compute the equations of motion in \ref{ex:2-1} again, but this time using Lagrange
multipliers.
\end{example}
\begin{solution}
Now we treat $x$ \& $y$ as independent variables, and they subject to $x^2+y^2-l^2=0$.
\begin{equation}
G\left(x, y\right) = x^2 + y^2 - l^2
\end{equation}
\begin{equation}
L = \frac{1}{2} m \left(\dot{x} ^ 2 + \dot{y} ^ 2\right) - mgy
\end{equation}
\begin{equation}
L + \lambda G = \frac{1}{2} m \left(\dot{x} ^ 2 + \dot{y} ^ 2\right) - mgy + \lambda \left(x^2 + y^2 - l^2\right)
\end{equation}
So by taking the derivative of $L + \lambda G$ with respect to $x$, we can get:
\begin{align}
\label{eq:4-12}
m \ddot{x} - 2 \lambda x &= 0 \\
\label{eq:4-13}
m \ddot{y} + mg - 2 \lambda y &= 0 \\
x ^ 2 + y ^ 2 &= l ^ 2
\end{align}
From \ref{eq:4-12} and \ref{eq:4-13} we get:
\begin{equation}
m \frac{\ddot{x}}{x} = m \left(\frac{\ddot{x}}{x} + \frac{g}{y}\right)
\end{equation}
\begin{equation}
\label{eq:4-16}
\ddot{x} y - \ddot{y} x - gx = 0
\end{equation}
Previously from \ref{eq:2-12} and \ref{eq:2-13} we get the time derivatives of the
coordinates are:
\begin{align}
\dot{x} &= l \cos{\theta} \cdot \dot{\theta} \\
\dot{y} &= l \sin{\theta} \cdot \dot{\theta}
\end{align}
So the second time derivatives of the coordinates are:
\begin{align}
\label{eq:4-19}
\ddot{x} &= l\left(\cos \theta \ \ddot{\theta} - \sin \theta \ \dot{\theta}^2\right) \\
\label{eq:4-20}
\ddot{y} &= l\left(\sin \theta \ \ddot{\theta} + \cos \theta \ \dot{\theta}^2\right)
\end{align}
Take \ref{eq:4-19} and \ref{eq:4-20} into \ref{eq:4-16}, we can get:
\end{solution}